Cardiology 2 Flashcards

1
Q
  1. B.C., a 40-year-old woman (height, 65 inches; weight,
    75 kg), presents to the emergency department (ED)
    with shortness of breath and lower extremity swelling
    in her left leg. Her diagnosis is a new acute pulmonary
    embolism, which was identified on chest computed
    tomography. Her medical history includes deep vein
    thrombosis (DVT) 2 years ago (treated with anticoagulation therapy for 3 months and then stopped),
    factor V Leiden mutation, hypertension (HTN), and
    dyslipidemia. Her medications are lisinopril 40 mg
    orally daily, atorvastatin 10 mg orally daily, and aspirin 81 mg orally daily. Initial laboratory values include
    hematocrit (HCT) 40% (normal 42%–52%), prothrombin time (PT) 9.9 seconds (normal 9.9–11.2 seconds),
    international normalized ratio (INR) 1.1, activated
    partial thromboplastin time (aPTT) 30 seconds (normal 24–36 seconds), platelet (Plt) count 300,000/
    mm3
    (normal 150,000–300,000/mm3
    ), and creatinine
    clearance (CrCl) 67 mL/minute. How long should this
    patient be treated with anticoagulation therapy?
    A. 3 months.
    B. 6 months.
    C. 12 months.
    D. Indefinitely.
  2. What is the most appropriate choice for anticoagulation for this patient?
    A. Apixaban 5 mg orally twice daily.
    B. Rivaroxaban 15 mg orally twice daily for 3 weeks
    then 20 mg orally daily thereafter.
    C. Enoxaparin 80 mg subcutaneous twice daily continued until INR is 2–3 with warfarin 5 mg orally
    daily and titrated to goal INR range.
    D. Aspirin 81 mg plus clopidogrel 75 mg orally daily.
A
  1. Answer: D
    With this being a recurrent VTE event and known factor V
    Leiden deficiency, life-long therapy with anticoagulation is
    indicated, making answer D correct. Answers A, B, and C
    are incorrect because they are shorter lengths of treatment,
    which is not the best because she has known a factor V
    Leiden mutation and now has had two VTE events warranting life-long treatment.
  2. Answer: B
    Answer A is incorrect because the dosing is not appropriate
    for the treatment of a VTE. Answer B is correct because
    the CHEST guidelines recommend use of a DOAC over
    a VKA to treat a VTE, despite fewer data with DOACs
    than with warfarin in patients with thrombophilias like
    factor V Leiden mutation. Answer C is incorrect because
    DOACs are preferred to VKAs to treat VTE according to
    the CHEST guidelines, even though this is correct dosing
    for LMWH and warfarin. Aspirin plus clopidogrel is not an
    anticoagulant therapy, making answer D incorrect
How well did you know this?
1
Not at all
2
3
4
5
Perfectly
2
Q
  1. B.G. is a 36-year-old man with a medical history
    of dyslipidemia, hypertension, chronic obstructive
    pulmonary disease (COPD), and heart failure with
    reduced ejection fraction. His blood pressure today in
    clinic is 128/80 mm Hg with a heart rate of 60 beats/
    minute. His current medication regimen includes furosemide 20 mg orally once daily, metoprolol succinate
    200 mg orally once daily, atorvastatin 20 mg orally
    once daily, tiotropium one 18-mcg inhalation once
    daily, and amlodipine 5 mg orally once daily. He has
    a history of lisinopril-induced angioedema. Which of
    the following is the best modification to his medication regimen based on the American Heart Association
    (AHA)/American College of Cardiology (ACC)/Heart
    Failure Society of America (HFSA) heart failure
    guideline recommendations?
    A. Add valsartan 80 mg orally daily.
    B. Add eplerenone 50 mg orally daily.
    C. Change furosemide to bumetanide 2 mg orally
    daily.
    D. Change metoprolol succinate to carvedilol 25 mg
    orally twice daily
A
  1. Answer: A
    An angiotensin-converting enzyme (ACE) inhibitor or
    ARB would be the next indicated heart failure medication,
    based on the ACCF/AHA heart failure guideline recommendations and clinical trial results. Starting the ARB
    valsartan 80 mg daily is the best choice for B.G. because
    of his lisinopril-induced angioedema, making answer A
    correct. Answer B is not correct because starting the aldosterone antagonist eplerenone is not appropriate for B.G., as
    he should be taking an ACE inhibitor or an ARB in addition to the β-blocker metoprolol succinate before adding
    an aldosterone antagonist. Answer C is not correct because
    hanging furosemide to bumetanide is not appropriate, as
    there is no evidence that B.G. is having issues with edema.
    Answer D is not correct because changing metoprolol to
    carvedilol is not appropriate, as there is no evidence to
    support a preference for the use of carvedilol instead of
    metoprolol succinate for the chronic management of heart
    failure based on morbidity or mortality
How well did you know this?
1
Not at all
2
3
4
5
Perfectly
3
Q
  1. P.Y. is a 57-year-old woman who states that she has
    had increased swelling in her legs over the last 2 weeks
    and increased shortness of breath in the last week.
    On physical examination, she has 2+ pitting edema
    bilaterally and no abnormal lung findings. She has
    a medical history of hypertension, heart failure with
    reduced ejection fraction (HFrEF), gout, hypothyroidism, and dyslipidemia. Her blood pressure today in
    clinic is 170/96 mm Hg with a heart rate of 76 beats
    per minute. Which of the following is the most likely
    cause of her symptoms in clinic today?
    A. Uncontrolled hypertension.
    B. Exacerbation of heart failure.
    C. Acute myocardial infarction.
    D. Peripheral artery disease.
A

. Answer: B
Answer B is correct because exacerbation of heart failure
is based on her history of heart failure and her presentation
with fluid in her lungs and lower extremity edema. Answer
A is incorrect because the patient would not develop pulmonary edema from elevation in blood pressure unless she
was presenting with extreme blood pressure elevation and
hypertensive crisis. Her blood pressure today (170/96 mm
Hg) is not extremely elevated to be consistent with hypertensive crisis. Answer C is incorrect because the patient’s
presentation is not consistent with an acute myocardial
infarction and because her symptoms have been occurring
for 1–2 weeks and no chest pain is described. Answer D
is incorrect because the patient does not have a history of
peripheral artery disease and she is not reporting the pain
that is typical in patients with peripheral artery disease

How well did you know this?
1
Not at all
2
3
4
5
Perfectly
4
Q
  1. R.R. is a 62-year-old woman with congestive heart
    failure is taking furosemide 80 mg orally twice daily,
    lisinopril 40 mg orally once daily, and metoprolol
    succinate 100 mg orally once daily. At her last clinic
    visit, her serum creatinine had increased to 2.0 mg/
    dL (her serum creatinine was 1.5 mg/dL 1 year ago).
    This change was accompanied by a 12-lb weight loss
    leaving her 5 lb less than her typical dry weight. The
    patient states that she feels well. Which of the following treatment options is best for this patient?
    A. Discontinue lisinopril and furosemide and recheck
    labs in 1 week.
    B. Discontinue lisinopril and recheck labs in 1 week.
    C. Discontinue furosemide and recheck labs in 1
    week.
    D. Continue current therapy and recheck labs in 1
    week
A
  1. Answer: C
    Answer C is correct because discontinuing furosemide
    and rechecking laboratory values in 1 week is the correct
    answer, and because the patient’s weight loss and increase
    in serum creatinine probably are the result of excessive
    diuresis. Her serum creatinine should decrease after stopping furosemide. Furosemide can be resumed at a future
    visit if the patient shows signs or symptoms of fluid overload, but a lower dose should be used at that time. Answer A
    is incorrect because lisinopril might be contributing to the
    rise in serum creatinine, but the drug seldom needs to be
    discontinued in this setting. Answer B is incorrect because
    the patient’s recent weight loss and increase in serum creatinine are the result of excessive diuresis and likely not
    related to her use of lisinopril. Answer D is incorrect
    because continuing current therapy and rechecking laboratory values in 1 week is incorrect, and the patient’s recent
    weight loss and increase in serum creatinine are the result
    of excessive diuresis.
How well did you know this?
1
Not at all
2
3
4
5
Perfectly
5
Q
  1. R.R. is a 65-year-old white woman with a medical history of type 2 diabetes mellitus, dyslipidemia, atrial
    fibrillation (AF), and gastroesophageal reflux disease
    (GERD) presents for comprehensive medication management. Her AF was diagnosed about 2 years ago,
    and has been titrated to up to metoprolol succinate
    200 mg daily. She currently has a blood pressure of
    140/90 mm Hg and a heart rate of 98 beats/minute.
    She reports that she is still experiencing symptoms of
    AF with palpitations almost daily despite taking her
    metoprolol each day. Which one of the following target heart rates is best to recommend as a rate-control
    strategy for this patient?
    A. Less than 150 beats/minute.
    B. Less than 110 beats/minute.
    C. Less than 80 beats/minute.
    D. Less than 60 beats/minute.
A
  1. Answer: C
    The ACC/AHA guidelines recommend targeting a heart
    rate goal of less than 110 beats/minute as the initial target
    and then the more aggressive target of less than 80 beats/
    minute if the patient is still symptomatic at the higher goal
    (answer C is correct). The guidelines do not advocate a
    heart target of less than 150 beats/minute or less than 60
    beats/minute (answers A and D are incorrect). The guidelines recommend that the more aggressive target of less
    than 80 beats/minute can be used if the patient is still
    symptomatic at the higher goal of less than 110 beats/minute (Answer B is incorrect).
How well did you know this?
1
Not at all
2
3
4
5
Perfectly
6
Q
  1. T.T. is a 68-year-old woman who presents to the
    cardiology clinic for a routine follow-up. Her medical
    history is significant for HTN, dyslipidemia, a transient
    ischemic attack (TIA) 2 weeks ago, paroxysmal AF,
    and HFrEF with an ejection fraction (EF) of 30%.
    Implantable cardioverter-defibrillator interrogations
    over the past year reveal that she has remained in
    sinus rhythm. Her current cardiac medications include
    aspirin 81 mg daily, metoprolol succinate 100 mg
    daily, atorvastatin 40 mg daily, losartan 50 mg daily,
    digoxin 125 mcg daily, amiodarone 200 mg daily, and
    apixaban 5 mg twice daily. Her vital signs in the clinic
    are blood pressure 105/70 mm Hg and heart rate 62
    beats/minute. Today, as you review her medications,
    the patient says that she wishes she did not have to take
    so many medications. Which one of the following is
    best to recommend for discontinuation in this patient?
    A. Aspirin.
    B. Digoxin.
    C. Amiodarone.
A
  1. Answer: B
    This patient has been in sinus rhythm for the past 12
    months, and her current heart rate is within goal; therefore,
    Cardiology II
    ACCP/ASHP 2023 Ambulatory Care Pharmacy Preparatory Review and Recertification Course
    292
    digoxin would be the most appropriate medication to discontinue at this time (answer B is correct). Discontinuing
    digoxin would also avoid a drug–drug interaction with
    amiodarone, which should not be discontinued because the
    patient is currently in sinus rhythm (answer C is incorrect).
    Given her recent transient ischemic attack, discontinuing
    aspirin at this time would not be the best option, but this
    could be considered in the future (answer A is incorrect).
    Metoprolol should not be discontinued because the patient
    has HFrEF (answer D is incorrect).
How well did you know this?
1
Not at all
2
3
4
5
Perfectly
7
Q
  1. F.C. is a 72-year-old woman (weight 51 kg) who presents to the ED with tachycardia and shortness of breath
    and receives a diagnosis of AF. Her medical history
    is notable for HTN, type 2 diabetes, GERD, coronary
    artery disease (CAD), hyperlipidemia, and COPD.
    Her serum creatinine (SCr) is 0.6 mg/dL. Which one
    of the following is best to recommend for this patient’s
    stroke prevention?
    A. Edoxaban.
    B. Rivaroxaban.
    C. Dabigatran.
    D. Warfarin.
A
  1. Answer: A
    The ENGAGE AF-TIMI subanalysis examined 8474
    patients older than 75 years to evaluate the risk-benefit
    of edoxaban in older adults. No significant interaction
    occurred between the treatment groups (edoxaban 30 mg
    vs. warfarin or edoxaban 60 mg vs. warfarin) and subgroups defined according to age younger than 75 or 75
    and older in subgroup analysis of ENGAGE AF-TIMI 48
    for the primary efficacy end point of stroke or systemic
    embolism, or for the primary safety end point of major
    bleeding, making answer A correct. In the subanalysis of
    the ROCKET-AF and RE-LY trials, no interaction was
    noted for age, but these agents were associated with an
    increase in GI bleeding, making answers B and C incorrect. DOACs are recommended over warfarin, making
    answer D incorrect
How well did you know this?
1
Not at all
2
3
4
5
Perfectly
8
Q
  1. S.G. is a 54-year-old male patient who presents to the
    ED with a chief complaint of intermittent palpitations
    and shortness of breath for the past 3 weeks. The initial
    assessment reveals a blood pressure of 154/88 mmHg
    and heart rate 86 bpm, and an electrocardiogram
    (ECG) reveals atrial fibrillation. The team is considering synchronized cardioversion for this patient. Which
    of the following treatment recommendations is most
    appropriate for this patient at this time?
    A. Anticoagulation is not needed before cardioversion, but it should be followed by 4 weeks of
    anticoagulation with dose-adjusted warfarin.
    B. Start treatment-dose heparin immediately, but
    delay cardioversion by 48 hours, followed by 4
    weeks of anticoagulation with dose-adjusted warfarin after cardioversion.
    C. Start treatment-dose heparin immediately and
    proceed with cardioversion now, followed by 4
    weeks of anticoagulation with dose-adjusted warfarin after cardioversion.
    D. Delay cardioversion, and begin anticoagulation
    with dose-adjusted warfarin for 3 weeks before
    and 4 weeks after cardioversion.
A
  1. Answer: D
    In a patient with a duration of atrial fibrillation greater
    than 48 hours or unknown, the preferred recommendation is to administer anticoagulation for at least 3 weeks
    before and for at least 4 weeks after cardioversion (answer
    D is correct). This treatment strategy would reduce the
    risk of stroke associated with cardioversion, although the
    success of cardioversion does decrease as the duration of
    atrial fibrillation increases. Treatment options for anticoagulation before cardioversion include warfarin (INR
    2–3), LMWH or UFH at treatment doses, dabigatran,
    rivaroxaban, or apixaban. It is important to note that this
    recommendation applies only to patients in which a delay
    in cardioversion would be acceptable. In those patients in
    need of emergent cardioversion (e.g., hemodynamically
    unstable), anticoagulation should not delay cardioversion.
    Finally, as noted above, a transesophageal echocardiogram can be performed before cardioversion to evaluate
    for thrombus and to guide the duration of anticoagulation
    therapy. Answers A, B, and C are incorrect
How well did you know this?
1
Not at all
2
3
4
5
Perfectly
9
Q
  1. P.L. is unresponsive to initiation of vasodilator treatment with a calcium channel blocker. The patient is
    classified as having pulmonary arterial hypertension
    (PAH) functional class (FC) II. Which is the best next
    combination therapy for treatment of PAH?
    A. Riociguat and tafalafil.
    B. Ambrisentan and tadalafil.
    C. Bosentan and sildenafil.
    D. Macitentan and sildenafil
A
  1. Answer: B
    Answer B is correct because the combination of ambrisentan and tadalafil are recommended for PAH FC II.
    Answers A, C, and D are incorrect because they are all
    recommended individually as monotherapy in PAH FC II,
    but they are not recommended as combination therapy
How well did you know this?
1
Not at all
2
3
4
5
Perfectly